2014-2015/2nd/DS/DC_02/DC_02.tex

424 lines
17 KiB
TeX
Raw Normal View History

2017-06-16 06:48:07 +00:00
\documentclass[a4paper,12pt, table]{/media/documents/Cours/Prof/Enseignements/2014-2015/tools/style/classExamen}
\usepackage{/media/documents/Cours/Prof/Enseignements/2014-2015/2014_2015}
\usepackage{csvsimple}
% Title Page
\titre{}
% \seconde \premiereS \PSTMG \TSTMG
\classe{Seconde}
\date{11 février 2015}
\typedoc{Devoir Commun}
\duree{3h}
\ptpres{4}
\printanswers
\begin{document}
\titlepage
\begin{questions}
\question[10]
% Depuis repère 67 p 157
Un magasin a annoncé sa journée de promotion par une distribution de tracts sur lesquels était indiqué:
\begin{center}
\textit{Grande journée de promotion! Dépensez moins!}
\end{center}
\textbf{Partie 1}\\
Le tableau ci-dessous donne les montants en euros, arrondis à l'unité, des achats effectués par les 80 clients du magasin pendant une journée ordinaire.
\definecolor{lightgray}{gray}{0.9}
\rowcolors{1}{lightgray}{}
\begin{center}
\begin{tabular}{|*{8}{c|}}
\hline
2 &3 &5 &5 &5 &8 &8 &8\\
\hline
8 &10 &10 &10 &10 &10 &10 &10\\
\hline
11 &13 &14 &14 &14 &20 &20 &20\\
\hline
20 &20 &20 &21 &24 &24 &25 &26\\
\hline
30 &30 &30 &30 &30 &30 &31 &33\\
\hline
33 &35 &36 &38 &38 &38 &38 &38\\
\hline
39 &39 &39 &39 &39 &40 &40 &40\\
\hline
40 &40 &40 &40 &40 &40 &42 &42\\
\hline
42 &43 &43 &43 &44 &44 &45 &45\\
\hline
45 &45 &45 &46 &46 &47 &55 &60\\
\hline
\end{tabular}
\end{center}
\begin{parts}
%1pt
\part Quelle est la population concernée par cette étude statistique? Quel est le caractère étudié?
\begin{solution}
Dans cette étude statistique, la population est l'ensemble des clients et le caractère étudié est leurs depenses.
\end{solution}
%1pts
\part Tracer le tableau des effectifs de cette série statistique.
\begin{solution}
Tableau des effectifs
\TODO{à faire..}
\end{solution}
% J'aimerai qu'ils aient à tracer un histogramme ici!
\part
\begin{subparts}
%1pt
\subpart Déterminer le pourcentage de clients ayant effectué des achats pour un montant ne dépassant pas les 27\euro.
\begin{solution}
Pourcentage des clients ayant fait des achats pour moins de 27\euro
\begin{eqnarray*}
\frac{32}{80} & = & 0,4 = 40\%
\end{eqnarray*}
\end{solution}
%1pt
\subpart Déterminer le pourcentage de clients ayant effectué des achats entre 30\euro\; et 40\euro\; inclus.
\begin{solution}
Pourcentage des clients ayant fait des achats entre 30 et 40\euro.
\begin{eqnarray*}
\frac{30}{80} & = & 0,375 = 37,5\%
\end{eqnarray*}
\end{solution}
\end{subparts}
% 2pt
\part Calculer la moyenne de cette série statistique.
\begin{solution}
\TODO{à faire..}
\end{solution}
\part
\begin{subparts}
%1pt
\subpart Déterminer le minimum et le maximum de cette série statistique.
\begin{solution}
En lisant le tableau de valeurs:
\begin{itemize}
\item Mininum: 2
\item Maximum: 60
\end{itemize}
\end{solution}
%2pts
\subpart Déterminer la médiane de cette série statistique.
\begin{solution}
Médiane de cette série. Dans le sujet les données sont déjà rangées par ordre croissant.
Effectif total: 80
Position de la médiane: $\frac{80}{2} = 40$ Donc la médiane se trouver entre la 40 et la 41ième valeur. Donc $Me =33 $.
\end{solution}
%2pts
\subpart Déterminer les quartiles de cette série statistique.
\begin{solution}
Position du premier quartile: $\frac{1}{4} \times 80 = 20$. Donc le premier quartile se trouve entre la 20ième et la 21ième valeur. Donc $Q_1 = 14$.
Position du troisième quartile: $\frac{3}{4} \times 80 = 60$. Donc le troisième quartile se trouve entre la 60ième et la 61ième valeur. Donc $Q_3 = 40$.
\end{solution}
\end{subparts}
\begin{EnvFullwidth}
\textbf{Partie 2}\\
Un étude similaire a été faite sur 80 clients lors d'une journée de promotion. Cette étude a donné les résultats suivants:
\begin{itemize}
\item Moyenne: 50
\item Minimum: 5
\item Premier quartile: 45
\item Médiane: 55
\item Troisième quartile: 63
\item Maximum: 75
\end{itemize}
\end{EnvFullwidth}
%2pts
\part En utilisant les résultats des deux études statistiques, commentez le message publicitaire de ce magasin.
\begin{solution}
On remarque que mis à part le minimum, tous les indicateurs ($Q_1$, $Me$, $Q_3$ et max) sont supérieurs lors d'une journée de promotion que lors d'une journée ordinaire. Les clients dépensent donc plus un jour de promotion qu'un jour ordinaire. Le slogan n'est donc pas véridique.
\end{solution}
\end{parts}
%\pagebreak
\question[9]
Dans un plan muni d'un repère orthonormé $(O;I;J)$, on a placé les points
\begin{eqnarray*}
T(-2;-2) \qquad R(0;2) \qquad I(2;1)
\end{eqnarray*}
\begin{parts}
%1pt
\part Faire la figure et la remplir au fil des questions suivantes.
\begin{solution}
\begin{center}
\begin{tikzpicture}[scale = 0.7]
\draw (-5,-5) grid (5,5);
\repere{-5}{5}{-5}{5}
\draw (-2,-2) node {$\bullet$} node [above right] {$T$};
\draw (0,2) node {$\bullet$} node [above right] {$R$};
\draw (2,1) node {$\bullet$} node [above right] {$I$};
\draw (-2,-2) -- (0,2) -- (2,1) -- (-2,-2);
\draw (0,-0.5) node {$\bullet$} node [below left] {$A$};
\draw (-1,0) node {$\bullet$} node [above left] {$0$};
\draw (-1,0) circle (2.23);
\end{tikzpicture}
\end{center}
\end{solution}
%2pts
\part Calculer les longueurs des trois côtés du triangle $TRI$.
\begin{solution}
Longueur du segment $[TR]$
\begin{eqnarray*}
TR & = & \sqrt{(x_T - x_R)^2 + (y_T - y_R)^2}\\
TR &=& \sqrt{(-2 - 0)^2 + (-2 - 2)^2}\\
TR &=& \sqrt{4 + 16}\\
TR &=& \sqrt{20}
\end{eqnarray*}
Longueur du segment $[TI]$
\begin{eqnarray*}
TI & = & \sqrt{(x_T - x_I)^2 + (y_T - y_I)^2}\\
TI &=& \sqrt{(-2 - 2)^2 + (-2 - 1)^2}\\
TI &=& \sqrt{16 + 9}\\
TI &=& \sqrt{25} = 5
\end{eqnarray*}
Longueur du segment $[RI]$
\begin{eqnarray*}
RI & = & \sqrt{(x_R - x_I)^2 + (y_R - y_I)^2}\\
RI &=& \sqrt{(0 - 2)^2 + (2 - 1)^2}\\
RI &=& \sqrt{4 + 1}\\
RI &=& \sqrt{5} \approx 2,23
\end{eqnarray*}
\end{solution}
%2pts
\part Démontrer que le triangle $TRI$ est un triangle rectangle. Est-il isocèle?
\begin{solution}
Pour savoir si le triangle est rectangle, on veut vérifier sur $TR^2 + RI^2$ est égal à $TI^2$.
\begin{eqnarray*}
TI^2 & = & 5^2 = 25 \\
TR^2 + RI^2 & = & \sqrt{20}^2 + \sqrt{5}^2 = 20 + 5 = 25
\end{eqnarray*}
Donc on a bien $TI^2 = TR^2 + RI^2$, d'après le théorème de Pythagore, le triangle $TRI$ est rectangle en $R$.
Ce triangle n'est pas isocèle, car d'après la question précédente, il y a aucun coté qui fait la même longueur qu'un autre.
\end{solution}
%2pts
\part Calculer les coordonnées du point $A$ milieu du segment $[TI]$. Placer ce point sur le dessin.
\begin{solution}
Calcul des coordonnées du point $A$ milieu de $[TI]$.
\begin{eqnarray*}
x_A & = & \frac{x_T + x_I}{2} = \frac{-2 + 2}{2} = 0 \\
y_A & = & \frac{y_T + y_I}{2} = \frac{-2 + 1}{2} = \frac{-1}{2} \\
\end{eqnarray*}
Donc $A (0;\frac{1}{2})$.
\end{solution}
\part
\begin{subparts}
% 0,5pt
\subpart Tracer le cercle $\mathcal{C}$ de diamètre $[TR]$.
\begin{solution}
Voir graphique
\end{solution}
% 1pt
\subpart Calculer les coordonnées de son centre.
\begin{solution}
Le centre du cercle, on l'appelle $O$, est le milieu de $[TR]$. On calcule ses coordonnées
\begin{eqnarray*}
x_0 & = & \frac{x_T + x_R}{2} = \frac{-2 + 0}{2} = -1 \\
y_0 & = & \frac{y_T + y_R}{2} = \frac{-2 + 2}{2} = 0
\end{eqnarray*}
\end{solution}
% 0,5
\subpart Calculer la mesure $r$ de son rayon.
\begin{solution}
Le rayon d'un cercle est égal à la moitier du diamètre $TR$. Donc
\begin{eqnarray*}
r & = & \frac{TR}{2} = \frac{\sqrt{20}}{2} \approx 2,23
\end{eqnarray*}
\end{solution}
\end{subparts}
\end{parts}
\pagebreak
\question[5]
Placer les points suivants sur le plan ci-dessous en laissant les traits de construction.
\begin{center}
\begin{tikzpicture}[scale=1]
\coordinate (A) at (4,5);
\coordinate (B) at (5,7);
\coordinate (C) at (6,4);
\coordinate (D) at (3,7);
\draw (0,0) grid (10,10);
\draw (A) node {$\bullet$} node[below left] {$A$};
\draw (B) node {$\bullet$} node[below right] {$B$};
\draw (C) node {$\bullet$} node[above right] {$C$};
\draw (D) node {$\bullet$} node[below left] {$D$};
\draw[very thick, ->] (2,7) -- (2,5) node[midway, above left] {$\vec{u}$};
\ifprintanswers
\draw[color = blue, very thick, ->] (B) -- (C) node[font=\tiny, sloped, midway, above] {$\vec{BC}$};
\draw[color = blue, very thick, ->] (A) --+ (1,-3) node[font=\tiny,sloped, midway, above] {$\vec{AU} = \vec{BC}$};
\draw[color = blue] (5,2) node {$\bullet$} node[below left] {$U$};
\draw[color = red, very thick, ->] (C) -- (D) node[font=\tiny,sloped, midway, above] {$\vec{CD}$};
\draw[color = red, very thick, <-] (A) --+ (3,-3) node[font=\tiny,sloped, midway, above] {$\vec{VA} = \vec{CD}$};
\draw[color = red] (7,2) node {$\bullet$} node[above right] {$V$};
\draw[color = green, very thick, ->] (A) -- (C) node[font=\tiny,sloped, midway, above] {$\vec{AC}$};
\draw[color = green, very thick, ->] (B) --+ (-2,1) node[font=\tiny,sloped, midway, above] {$\vec{BW} = -\vec{AC}$};
\draw[color = green] (3,8) node {$\bullet$} node[below left] {$W$};
\draw[color = purple, very thick, ->] (C) --+ (1,-3);
\draw[color = purple, very thick, ->] (7,1) --+ (-3,3);
\draw[color = purple] (4,4) node {$\bullet$} node[below left] {$X$};
\fi
\end{tikzpicture}
\end{center}
\begin{parts}
\part Le point $U$ tel que $\vec{AU} = \vec{BC}$.
\begin{solution}
En bleu
\end{solution}
\part Le point $V$ tel que $\vec{VA} = \vec{CD}$.
\begin{solution}
En rouge
\end{solution}
\part Le point $W$ tel que $\vec{BW} = -\vec{AC}$.
\begin{solution}
En vert
\end{solution}
\part Le point $X$ tel que $\vec{CZ} = \vec{BC} + \vec{CD}$.
\begin{solution}
En violet
\end{solution}
\part Est-il vrai que $2\vec{u} + \vec{BA} + \vec{DA} = \vec{0}$? \textit{La justification peut être un dessin ici.}
\begin{solution}
\TODO{à faire}
\end{solution}
\end{parts}
\pagebreak
\question[6]
\begin{parts}
\part Voici le tableau de variation de la fonction $f$
\begin{tikzpicture}
\tkzTabInit[espcl=2]{$x$/1,$f(x)$/1}{-4, -2, 0, 1, 2, 4}
\tkzTabVar{-/{-4}, +/{1}, -/{-3}, +/{0}, -/{-3}, +/{3}}
\end{tikzpicture}
\begin{subparts}
%1pt
\subpart Sur quels intervalles la fonction $f$ est-elle décroissante?
\begin{solution}
D'après le tableau de variations, la fonction $f$ est décroissante sur $\intFF{-2}{0} \cup \intFF{1}{2}$.
\end{solution}
%1pt
\subpart Déterminer le maximum de la fonction sur l'intervalle $\intFF{-4}{4}$.
\begin{solution}
Sur l'intervalle $_intFF{-4}{4}$ (l'intervalle de définition de $f$), le maximum est atteint pour $x = 4$ et vaut $f(4) = 3$.
\end{solution}
%1pt
\subpart Tracer une fonction qui a ce tableau de variation.
\begin{solution}
Voici une courbe possible
\begin{center}
\begin{tikzpicture}[scale=0.8]
\repere{-6}{6}{-6}{6}
\draw[very thick, color=red] plot [smooth,tension=0.5, mark=*] coordinates{%
(-4, -4) (-2,1) (0,-3) (1,0) (2,-3) (4,3)
};
\draw (4,3) node[above right] {$\mathcal{C}_f$};
\end{tikzpicture}
\end{center}
\end{solution}
\end{subparts}
\part Voici la représentation graphique de la fonction $g$.
\begin{center}
\begin{tikzpicture}[scale=0.8]
\repere{-6}{6}{-6}{6}
\draw[very thick, color=red] plot [smooth,tension=0.5, mark=*] coordinates{%
(-4, -4) (-3.5, 0) (-3, 4) (-2, 1) (-1, 0) (0, -1) (1, -2) (2, 1) (3, 3) (4, 2)%
};
\draw (4,3) node[above right] {$\mathcal{C}_g$};
\ifprintanswers
\draw[color = blue, dashed, very thick] (3,0) node[below] {3} -- (3,3) -- (0,3) node[left] {$f(3)$};
\draw[color=green, dashed, very thick] (-6,1) -- (6,1) node [above] {$y=1$};
\draw[color=green, dashed, very thick] (-3.4,1) node {$\bullet$} -- (-3.4,0) node [below] {-3,4};
\draw[color=green, dashed, very thick] (-2,1) node {$\bullet$} -- (-2,0) node [below] {-2};
\draw[color=green, dashed, very thick] (2,1) node {$\bullet$} -- (2,0) node [below] {2};
\fi
\end{tikzpicture}
\end{center}
\begin{subparts}
% O.5pt
\subpart Quel est l'image de 3 par cette fonction? Vous laisserez les traits de construction qui vous ont permis de répondre.
\begin{solution}
L'image de 3 par la fonction $g$ est 3. Voir les traits en bleu.
\end{solution}
% O.5pt
\subpart Quels sont les antécédents de 1 par cette fonction? Vous laisserez les traits de construction qui vous ont permis de répondre.
\begin{solution}
Les antécédents de 1 par cette fonction sont 2, -2 et environ -3,4. Voir les traits en vert.
\end{solution}
%2pts
\subpart Tracer le tableau variation de $f$.
\begin{solution}
Tableau de variation de cette fonction
\begin{tikzpicture}
\tkzTabInit[espcl=2]{$x$/1,$f(x)$/1}{-4, -3, 1, 3, 4}
\tkzTabVar{-/{-4}, +/{4}, -/{-2}, +/{3}, -/{2}, }
\end{tikzpicture}
\end{solution}
\end{subparts}
\end{parts}
\pagebreak
\question[6]
Cet exercice est une questionnaire à choix multiplies (QCM).
Dans le tableau ci dessous, \textbf{entourer} une réponse V (vrai) ou F (faux) à droite de chaque affirmation, en vous référant aux données situées dans la colonne de gauche.
\begin{itshape}
Une bonne réponse rapporte 0,5 points et une mauvaise réponse retire 0,25 point. L'absence de réponse ne donne ni n'enlève de point.
\end{itshape}
\hspace{-2cm}
\ifprintanswers
\includegraphics[scale=0.75]{./fig/QCM_coor}
\else
\includegraphics[scale=0.75]{./fig/QCM}
\fi
\end{questions}
\end{document}
%%% Local Variables:
%%% mode: latex
%%% TeX-master: "master"
%%% End: